0% found this document useful (0 votes)
27 views5 pages

Handout Andvaced NTTechniques Solutions

Uploaded by

lcm.jimenez.pers
Copyright
© © All Rights Reserved
We take content rights seriously. If you suspect this is your content, claim it here.
Available Formats
Download as PDF, TXT or read online on Scribd
0% found this document useful (0 votes)
27 views5 pages

Handout Andvaced NTTechniques Solutions

Uploaded by

lcm.jimenez.pers
Copyright
© © All Rights Reserved
We take content rights seriously. If you suspect this is your content, claim it here.
Available Formats
Download as PDF, TXT or read online on Scribd
You are on page 1/ 5

Advanced Number Theory Techniques Solutions

Chinese Remainder Theorem


1. Prove that there are n consecutive positive integers, none of which are the sum of two perfect squares.
Solution: We begin with several results that may be considered classical.
First of all, it would be nice to have some way to construct integers that are not sums of squares. Let p be any
odd prime. Notice that if x2 + y 2 ≡ 0 (mod p) then either p | x and p | y or x and y are invertible modulo p. Let
y −1 be some integer such that yy −1 ≡ 1 (mod p). Then multiplying by (y −1 )2 gives
(y −1 )2 (x2 + y 2 ) ≡ (xy −1 )2 + 1 (mod p).
This means that if p divides x2 + y 2 for any integers x and y with p ̸| x, y then there exists a solution to
u2 ≡ −1 (mod p).
Now Fermat’s Little Theorem tells us that up−1 ≡ 1 (mod p) so
(−1)(p−1)/2 ≡ 1 (mod p).
This means that p−12 is even so p ≡ 1 (mod 4). Therefore if p = 4k + 3 for some k then x + y can only be
2 2

divisible by p when p | x, y.
Notice furthermore that if x and y are divisible by p = 4k + 3 then p2 | x2 + y 2 . This means that if N is any
number divisible by p but not p2 , we know N cannot be written as a sum of squares.
Next, we prove there are infinitely many primes of the form 4k + 3. Suppose there are finitely many such
primes, and let them be p1 , p2 , . . . , pm . Then one of p1 p2 · · · pm + 2 or p1 p2 · · · pm + 4 is equivalent to 3 modulo
4 and is not divisible by any of the primes listed. But such a number must be divisible by some other prime of
the form 4k + 3.
We can combine the previous results with the CRT to find our sequence. Choose n primes, p0 , . . . , pn−1 , each
of which is congruent to 3 modulo 4. Consider the system
x + 0 ≡ p0 (mod p20 )
x + 1 ≡ p1 (mod p21 )
..
.
x + n − 1 ≡ pn−1 (mod p2n−1 ).

There exists some solution x to this system by the CRT. Since each x + j is divisible by some p = 4k + 3 but
not p2 we know that x + j cannot be written as a sum of two squares.

Vieta Jumping
1. Let a and b be positive integers. Show that if 4ab − 1 divides (4a2 − 1)2 , then a = b.
Solution: Since 4ab − 1 and b are relatively prime, 4ab − 1 divides (4a2 − 1)2 if and only if 4ab − 1 divides
b2 (4a2 − 1)2 . Expanding, that is
4ab − 1 | 16a4 b2 − 8b2 a2 + b2 = (4ab − 1) 4a3 b − 2ab + a2 + (a − b)2 .


Thus we must have that 4ab − 1 divides (a − b)2 . Suppose towards a contradiction that a ̸= b. Let
(a − b)2
k= >0
4ab − 1

© 2023 AoPS Incorporated 1


Advanced Number Theory Techniques Solutions
(a−b)2
and assume without loss of generality that a + b is least such that 4ab−1 = k.
We consider the quadratic equation (x − b)2 = 4xbk − k, or

x2 − (2b + 4kb)x + (b2 + k) = 0.


2
Clearly a is a solution of this equation. By Vieta’s formulas the other solution to this equation is b a+k . By
2
minimality of a + b, we have b a+k ≥ a. Hence we conclude that k ≥ a2 − b2 = (a − b)(a + b). Therefore,
k > (a − b)2 . However, this is a contradiction since

(a − b)2
k= ≤ (a − b)2 .
4ab − 1

Lifting the Exponent


1. Prove that if p is an odd prime and x and y are coprime to p such that p | x − y then

vp (xn − y n ) = vp (x − y) + vp (n).

Solution: First notice that if x = y then both vp (xn − y n ) and vp (x − y) are infinite so the claim is true but
uninteresting.
We break this proof into steps.
Step 1: Assume n = p. Then

xp − y p = (x − y)(xp−1 + xp−2 y 1 + · · · + x1 y p−2 + y p−1 )

so vp (xp − y p ) = vp (x − y) + vp (xp−1 + xp−2 y 1 + · · · + x1 y p−2 + y p−1 ). We want to prove that this sum is
divisible by p but not by p2 . To that end, we consider the sum modulo p2 .
Since p | x − y we can write y = x + pz for some integer z. Then by the binomial theorem

y k ≡ (x + pz)k ≡ xk + kpzxk−1 (mod p2 ).

Therefore
p−1
X p−1
X
xp−k−1 y k ≡ xp−k−1 (x + pz)k
k=0 k=0
p−1
X
≡ xp−k−1 (xk + kpzxk−1 )
k=0
p−1
X p
X
≡ xp−1 + kpzxp−2
k=0 k=0
p−1
X
≡ pxp−1 + pzxp−2 k
k=0
p(p − 1)
≡ pxp−1 + pzxp−2
2
≡ pxp−1 2
(mod p ).

In the last step we note that for an odd prime p, p | p(p−1)


2 so the second summand is divisible by p2 . Since p
does not divide x, we find vp (xp−1 + xp−2 y 1 + · · · + x1 y p−2 + y p−1 ) = 1 and we are done with the n = p step.

© 2023 AoPS Incorporated 2


Advanced Number Theory Techniques Solutions
Step 2: Assume n and p are relatively prime. Now

vp (xn − y n ) = vp (x − y) + vp (xn−1 + xn−2 y 1 + · · · + x1 y n−2 + y n−1 ).


Pn−1
We want to show that p does not divide k=0 xn−k−1 y k . Now we know y ≡ x (mod p) so
n−1
X n−1
X
xn−k−1 y k ≡ xn−1 ≡ nxn−1 (mod p).
k=0 k=0

Since n is not divisible by p and x is not divisible by p,

vp (xn − y n ) = vp (x − y) + vp (xn−1 + xn−2 y 1 + · · · + x1 y n−2 + y n−1 ) = vp (x − y) = vp (x − y) + vp (n).

Note that we did not use that p was odd for this step.
Step 3: We prove the remainder of the result by induction on vp (n). The base case vp (n) = 0 is Step 2, so we
now assume p | n. We have vp (n) > 0, so we can define integers X and Y by X p = x and Y p = y. Let m = np
so X = xm and Y = y m . Fermat’s Little Theorem tells us that X − Y ≡ x − y ≡ 0 (mod p). Therefore by Step 1,

vp (xn − y n ) = vp (X p − Y p ) = vp (X − Y ) + 1 = vp (xm − y m ) + 1.

Since vp (m) = vp (n) − 1 we have by induction that

vp (xn − y n ) = vp (xm − y m ) + 1 = vp (x − y) + vp (m) + 1 = vp (x − y) + vp (n).

2. Find x and y odd and a positive integer n such that

v2 (xn − y n ) ̸= v2 (x − y) + v2 (n).

Solution: Let x = 3 and y = 1. Then


v2 (x2 − y 2 ) = v2 (8) = 3
but
v2 (x − y) + v2 (2) = v2 (2) + v2 (2) = 2.

3. Prove that if x and y are odd with 4 | x − y and n is a positive integer then

v2 (xn − y n ) = v2 (x − y) + v2 (n).

Solution: As in Exercise ??, Step 2, we know that if n is odd then v2 (xn − y n ) = v2 (x − y). Therefore if n = 2m k
then m m m m
v2 (xn − y n ) = v2 ((x2 )k − (y 2 )k ) = v2 (x2 − y 2 )
and v2 (n) = v2 (2m ). Therefore we may assume that n = 2m for some nonnegative m.
Now applying difference of squares m times gives
m m
 m−1 m−1
  m−1 m−1

x2 − y 2 = x2 + y2 x2 − y2
 m−1 m−1
  m−2 m−2
  m−2 m−2

= x2 + y2 x2 + y2 x2 − y2
..
.
 m−1 m−1
  m−2 m−2

= x2 + y2 x2 + y2 · · · x2 + y 2 (x + y) (x − y).


© 2023 AoPS Incorporated 3


Advanced Number Theory Techniques Solutions
Or more succinctly,
m−1
m m Y k k
x2 − y 2 = (x − y) (x2 + y 2 ).
k=0

k k
Since x and y are odd, for k positive, x2 ≡ y 2 ≡ 1 (mod 4) so their sum is even but not a multiple of 4.
Therefore if k > 0,
k k
v2 (x2 + y 2 ) = 1.
Therefore,
m−1
m m X k k
v2 (x2 − y 2 ) = v2 (x − y) + v2 (x2 + y 2 )
k=0
m−1
X
= v2 (x − y) + v2 (x + y) + 1
k=1
= v2 (x − y) + v2 (x + y) + m − 1.

Moreover, since 4 | x − y we know y = x + 4z for some z and v2 (x + y) = v2 (2x + 4z) = 1. Therefore


m m
v2 (x2 − y 2 ) = v2 (x − y) + 1 + m − 1 = v2 (x − y) + m = v2 (x − y) + v2 (n).

4. Prove that if x and y are odd and n is an even positive integer then

v2 (xn − y n ) = v2 (x − y) + v2 (x + y) + v2 (n) − 1.

Solution: This follows immediately from the work of Exercise ??.

© 2023 AoPS Incorporated 4


Art of Problem Solving is an ACS WASC Accredited School.
Thanks to our sponsors:

You might also like